LSAT Explanation PT 29, S4, Q24: Medical researcher: As expected, records covering
LSAT Question Stem
Which one of the following, if true, most weakens the medical researcher's argument?
Logical Reasoning Question Type
This is a Weaken question.
Correct Answer
The correct answer to this question is B.
LSAT Question Complete Explanation
The question type for this problem is Weaken, and we are asked to find the answer choice that most weakens the medical researcher's argument.
First, let's analyze the argument in the passage. The medical researcher presents two premises:
1. Babies born prematurely were more likely to have low birth weights and health problems than babies not born prematurely.
2. Mothers who received adequate prenatal care were less likely to have low birth weight babies than mothers who received inadequate prenatal care.
Based on these premises, the researcher concludes that adequate prenatal care significantly decreases the risk of low birth weight babies. In other words, there is a causal relationship between adequate prenatal care (PC) and decreased risk of low birth weight babies (DR): PC ‚Üí DR.
Now, let's evaluate each answer choice:
a) This answer choice states that many normal birth weight babies are born to mothers who had inadequate prenatal care. While this may be true, it does not weaken the argument that adequate prenatal care decreases the risk of low birth weight babies. It is still possible for mothers with inadequate prenatal care to have normal birth weight babies, but the argument is focused on the benefits of adequate prenatal care.
b) This answer choice weakens the argument by showing that there may be a statistical problem with the data used to make the causal statement. If hospitals routinely classify mothers giving birth prematurely as having received inadequate prenatal care when the record of that care is not available, this could lead to unreliable data. This undermines the conclusion that adequate prenatal care significantly decreases the risk of low birth weight babies, making it the correct answer.
c) This answer choice states that low birth weight babies were routinely classified as having been born prematurely. However, this does not affect the conclusion about the relationship between adequate prenatal care and the risk of low birth weight babies.
d) This answer choice suggests that some babies not born prematurely, whose mothers received adequate prenatal care, have low birth weights. Similar to answer choice (a), this possibility does not weaken the argument, as the likelihoods discussed in the passage still allow for this scenario.
e) This answer choice actually strengthens the argument by showing that women who receive adequate prenatal care are less likely to give birth prematurely than those who do not receive adequate prenatal care. This demonstrates a positive effect of adequate prenatal care.
In conclusion, the correct answer is (b), as it weakens the medical researcher's argument by highlighting a potential issue with the data used to make the causal statement.
